Относительная плотность по… задачи | Дистанционные уроки

24-Фев-2013 | комментариев 26 | Лолита Окольнова

В ЕГЭ иногда встречаются задачи (часть С последнее задание), где в условии дана относительная плотность вещества по… водороду, кислороду, воздуху, азоту и т.д.

 

Например:

 

 

 

 

Относительная плотность  — величина безразмерная

Формула достаточно простая, и из нее вытекает другая формула —

Формула молярной массы вещества

 

Mr1 = D•Mr2

 

 

и т.д.

 

В условии задачи может быть полная формулировка — «относительная плотность (паров)…», а может быть просто «плотность вещества по…»

 

Давайте решим нашу задачу:

 

 

Дана плотность паров вещества по воздуху, значит, нам подходит формула  молярной массы вещества —

 

Mr (вещества)=Mr(воздуха)•D=29 г\моль • D

 

Mr(вещества)=29 г\моль • 1. 448 = 42 г\моль

 

Нам дан углеводород — СхHy, значит, мы можем найти Mr(Cx и Mr(Hy). Обратите внимание, именно молярные массы, т.к.у нас несколько атомов углерода и водорода.

Для этого надо молярную массу вещества умножить на процентное содержание элемента:

 

Mr(Cx)=Mr(вещества)•ω

Mr(Cx)= 42 г\моль · 0.8571=36 г\моль

x=Mr(Cx)\Ar(C)=36 г\моль ÷ 12 г\моль =3.

Точно  так же находим все данные для водорода:

Mr(Hy)=Mr(вещества)•ω

Mr(Hy)= 42 г\моль · 0.1429=6 г\моль

x=Mr(Hy)\Ar(H)=6 г\моль ÷ 1 г\моль =6.

 

Искомое вещество — C3H6 — пропен.

 

Еще раз повторим определение —

 

Относительная плотность газа – это сравнение молярной или относительной молекулярной массы одного газа с аналогичным показателем другого газа.

 

 

Дана относительная плотность по аргону.

 

Mr (вещества)=Ar(Ar)•D

 

Mr (CxHy)=40 г\моль ·1.05=42 г\моль

 

Запишем уравнение горения:

 

СхHy + O2 = xCO2 + y\2h3O

 

Найдем количество углекислого газа и воды:

 

n(CO2)=V\22,4 л\моль = 33.6\22.4=1.5

 

n(h3O)=m\Mr=27\18=1.5

 

Соотношение х : y\2 как 1.5 : 1.5, т.е. y=2x, что соответствует общей формуле алкенов: Cnh3n

 
Выражаем в общем виде молярную массу: Mr=Mr(C) + Mr(H)
 

12n +2n=42

 

n=3

 

Наше вещество — C3H6 — пропен

 


  • pадание ЕГЭ по этой теме — задачи С5

 
 

Еще на эту тему:

Обсуждение: «Относительная плотность по… задачи»

(Правила комментирования)

Задачи на вывод молекулярной формулы. Iтип Относительная плотность паров предельного альдегида по воздуху равна Выведите молекулярную формулу альдегида

Задачи на вывод молекулярной формулы. I тип

  1. Относительная плотность паров предельного альдегида по воздуху равна 2. Выведите молекулярную формулу альдегида.

  2. Найдите молекулярную формулу гомолога метана, если известно, что масса 5,6 л его (н.у.) составляет 18 г.

  3. Относительная плотность паров предельного альдегида по азоту равна 3,07. Выведите молекулярную формулу альдегида.

  4. Относительная плотность паров циклоалкана по азоту равна 5. Выведите молекулярную формулу циклоалкана.

  5. Относительная плотность паров предельной двухосновной карбоновой кислоты по водороду равна 59. Выведите молекулярную формулу карбоновой кислоты.

  6. Относительная плотность паров алкена по воздуху равна 2,414. Выведите молекулярную формулу алкена.

  7. Относительная плотность паров предельного двухатомного спирта по угарному газу равна 2,214. Выведите молекулярную формулу спирта.

  8. Относительная плотность паров алкадиена по диоксиду углерода равна 2,5. Выведите молекулярную формулу алкадиена.

  9. Относительная плотность паров спиртокислоты по метану равна 5,625. Выведите её молекулярную формулу.

  10. Гомолог бензола имеет относительную плотность паров по воздуху 4,62. Выведите его молекулярную формулу.

Задачи на вывод молекулярной формулы. II тип

  1. Относительная плотность паров углеводорода по углекислому газу равна 2,41. Массовая доля водорода в нём равна 9,43%. Выведите молекулярную формулу углеводорода.

  2. Найдите молекулярную формулу углеводорода, массовая доля водорода в котором составляет 15,79%. Относительная плотность паров этого вещества по воздуху равна 3,93.

  3. Относительная плотность паров органического соединения по водороду равна 51. Массовая доля углерода в веществе равна 58,82%, массовая доля кислорода равна 31,38%. Выведите молекулярную формулу органического соединения.

  4. Плотность углеводорода при нормальных условиях равна 2,59г/л. Массовая доля углерода в нём равна 82,76%. Выведите молекулярную формулу этого соединения.

  5. Найдите молекулярную формулу углеводорода, массовая доля углерода в котором составляет 83,3%. Относительная плотность паров этого вещества по водороду равна 36.

  6. Относительная плотность паров углеводорода по азоту равна 3. Массовые доли углерода и водорода в нём равны соответственно 85,71% и 14,29%. Выведите молекулярную формулу углеводорода.

  7. Относительная плотность паров углеводорода по кислороду равна 3,75. Массовая доля углерода в нём равна 90%. Выведите молекулярную формулу углеводорода.

  8. Относительная плотность паров органического соединения по воздуху равна 2,552. Массовая доля углерода в веществе равна 64,86%, массовая доля водорода равна 13,51%, массовая доля кислорода равна 21,63%. Выведите молекулярную формулу этого вещества.

  9. Относительная плотность паров углеводорода по водороду равна 41. Массовая доля водорода в нём равна 12,2%. Выведите молекулярную формулу углеводорода.

  10. Относительная плотность паров углеводорода по водороду равна 55. Массовая доля углерода в нём равна 87,27%. Выведите молекулярную формулу углеводорода.

Задачи на вывод молекулярной формулы. III тип

  1. При сжигании углеводорода массой 3,2 г образовалось 9,9 г оксида углерода(IV) и 4,5 г воды. Относительная плотность этого вещества по водороду равна 64. Найдите молекулярную формулу углеводорода.

  2. Относительная плотность паров органического вещества по водороду равна 30. При сжигании 24 г этого вещества образовалось 35,2 г диоксида углерода и 14,4 г воды. Выведите молекулярную формулу органического соединения.

  3. Относительная плотность паров органического соединения по кислороду равна 4,25. При сжигании 13,6 г этого вещества образовалось 39,6 г диоксида углерода и 10,8 г воды. Выведите молекулярную формулу органического соединения.

  4. Относительная плотность органического соединения по кислороду равна 1,375. При сжигании 4,4 г этого вещества образуется 13,2 г диоксида углерода и 7,2 г воды. Выведите молекулярную формулу этого органического соединения.

  5. Относительная плотность паров органического вещества по воздуху равна 2,69. При сгорании 19,5 г этого вещества образовалось 66 г углекислого газа и 13,5 г воды. Выведите молекулярную формулу органического вещества.

  6. Относительная плотность паров органического вещества по воздуху равна 4,69. При сжигании 95,2 г этого вещества образовалось 154 г диоксида углерода и 75,6 г воды. Выведите молекулярную формулу органического вещества.

  7. Относительная плотность паров органического вещества по водороду равна 42. При сжигании 7г этого вещества образуется 22 г диоксида углерода и 9г воды. Выведите молекулярную формулу органического вещества.

  8. *1,875г органического соединения при нормальных условиях занимает объём 1 л. При сжигании 4,2 г этого соединения образуется 13,2 г углекислого газа и 5,4 г воды. Выведите молекулярную формулу органического соединения.

  9. При горении 1 моль газообразного алкана (при н.у.) образовалось 22,4 л оксида углерода(IV) и 36 г воды. Найдите молекулярную формулу алкана.

  10. При сжигании углеводорода массой 29 г образовалось 88 г оксида углерода(IV) и 45 г воды. Относительная плотность вещества по воздуху равна 2. Найдите молекулярную формулу углеводорода.

Установите молекулярную формулу углеводорода содержащего 85,7% углерода, если относительная плотность паров вещества по водороду равна 21.

Установление молекулярной формулы вещества по данным состава

Установите молекулярную формулу углеводорода содержащего 85,7% углерода, если относительная плотность паров вещества по водороду равна 21.
Дано:

·(С) = 85,7% или 0,857 СхНу
D(h3) = 2 Mr(в-ва) = D(h3)
· 2 = 21
· 2 = 42
М.Ф. = ?
Пусть вещества 110г, тогда масса углерода в нем m(C) = 85,7г,
а масса водорода m(Н) = 14,3г.

·(С) :
·(Н) = 13 EMBED Equation.3 14150,1 : 0,3 :0,05 = 2 : 6 : 1

Другой способ
масса углерода в моль вещества m(С) = 42
· 0,857 = 36
тогда число атомов углерода в молекуле равно 36 : 12 = 3,
Остальное водород 42 –36 = 6, атомов водорода 6.
Ответ: С3Н6
Другой способ
масса углерода в моль вещества m(С) = 42
· 0,857 = 36
Тогда число атомов углерода в молекуле равно 36 : 12 = 3,
Остальное водород 42 –36 = 6, атомов водорода 6.
Ответ: С3Н6
Другой способ
масса углерода в моль вещества m(С) = 42
· 0,857 = 36
тогда число атомов углерода в молекуле равно 36 : 12 = 3,
остальное водород 42 –36 = 6, атомов водорода 6.
Ответ: С3Н6
Другой способ
масса углерода в моль вещества m(С) = 42
· 0,857 = 36
тогда число атомов углерода в молекуле равно 36 : 12 = 3,
остальное водород 42 –36 = 6, атомов водорода 6.
Ответ: С3Н61515iв осуществляется через соединения типа оснований Шиффа:

В дальнейшем была предложена иная схема мутаротации через образование иммониевого иона [20]:

Этот механизм объяс
·
·
·
·
·
·
·
·
·
·
·
·
·
·
·
·
·
·
·
·
·
·
·
·
·
·
·
·
·
·
·
·
·
·
·
·
·
·
·
·
·
·
·
·
·
·
·
·
·
·
·
·
·
·
·
·
·
·
·
·
·
·
·
·
·
·
·
·
·
·
·
·
·
·
·
·
·
·
·
·
·
·
·
·
·
·
·
·
·
·
·
·
·
·
·
·
·
·
·
·
·
·
·
·
·
·
·
·
·
·
·
·
·
·
·
·
·
·
·
·
·
·
·
·
·
·
·
·
·
·
·
·
·
·
·
·
·
·
·
·
·
·
·
·
·
·
·
·
·
·
·
·
·
·
·
·
·
·
·
·
·
·
·
·
·
·
·
·
·
·
·
·
·
·
·
·
·
·
·
·
·
·
·
·
·
·
·
·
·
·
·
·
·
·
·
·
·
·
·
·
·
·
·
·
·
·
·
·
·
·
·
·
·
·
·
·
·
·
·
·
·
·
·
·
·
·
·
·
·
·
·
·
·
·
·
·
·
·
·
·
·
·
·
·
·
·
·
·
·
·
·
·
·
·
·
·
·
·
·
·
·
·
·
·
·
·
·
·
·
·
·
·
·
·
·
·
·
·
·
·
·
·
·
·
·
·
·
·
·
·
·
·
·
·
·
·
·
·
·
·
·
·
·
·
·
·
·
·
·
·
·
·
·
·
·
·
·
·
·
·
·
·
·
·
·
·
·
·
·
·
·
·
·
·
·
·
·
·
·
·
·
·
·
·
·
·
·
·
·
·
·
·
·
·
·
·
·
·
·
·
·
·
·
·
·
·
·
·
·
·
·
·
·
·
·
·
·
·
·
·
·
·
·
·
·
·
·
·
·
·
·
·
·
·
·
·
·
·
·
·
·
·
·
·
·
·
·
·
·
·
·
·
·
·
·
·
·
·
·
·
·
·
·
·
·
·
·
·
·
·
·
·
·
·
·
·
·
·
·
·
·
·
·
·
·
·
·
·
·
·
·
·
·
·
·
·
·
·
·
·
·
·
·
·
·
·
·
·
·
·
·
·
·
·
·
·
·
·
·
·
·
·
·
·
·
·
·
·
·
·
·
·
·
·
·
·
·
·
·
·
·
·
·
·
·
·
·
·
·
·
·
·
·
·
·
·
·
·
·
·
·
·
·
·
·
·
·

Приложенные файлы

  • 5777238
    Размер файла: 46 kB Загрузок: 0

Индивидуальные домашние задания «Теоретические основы органической химии.

Задачи на вывод формулы вещества»

 10 класс

Домашнее задание №1: «Теоретические основы органической химии.

Задачи на вывод формулы вещества»

ВАРИАНТ 1

Дайте определения следующим понятиям: размер атома, электроотрицательность. Расположите следующие группы атомов в порядке возрастания их размера, их электроотрицательности:

    a) O, N, C, F, Li, Na; б) S, P, N, Cl, Al, Si; в) Cl, Br, F, Se, As, O; г) С, Cl, Li, Be, N.

    Какой из вышеприведенных атомов имеет наибольшую, а какой наименьшую электроотрицатальность?


     

    Какие из нижеприведенных формул принадлежат алканам:

      C4H8, C5h22, C6h24, C8h28, C9h28, C10h32, C11h34, C12h34, C13h36, C15h40, C16h44.


       

      Относительная плотность паров алкана по водороду равна 57. Выведите молекулярную формулу алкана.

        Относительная плотность паров органического соединения по азоту равна 3,071. При сжигании 8,6 г этого вещества образуется 11,2 л углекислого газа и 9 мл воды (н.у.). Выведите молекулярную формулу этого соединения.

          10 класс

          Домашнее задание №1: «Теоретические основы органической химии.

          Задачи на вывод формулы вещества»

          ВАРИАНТ 2

          Укажите знак электронного эффекта для следующих заместителей: COOH, COO, C2H5, Ch4C=O, Br, Li, O, CN, Nh4, SH, HgC2H5, Si(Ch4)3, CH=O, =O, C(Ch4)3, S, OH. Ответы поясните.

            Напишите структурные формулы всех возможных изомерных алканов состава:

              а) C5h22; б) C6h24; в) C7h26; г) C8h28.

              Назовите эти вещества по международной номенклатуре. Укажите первичные, вторичные, третичные и четвертичные атомы углерода.

              Относительная плотность паров алкана по азоту равна 5,07. Выведите молекулярную формулу алкана.

                Относительная плотность паров органического соединения по водороду равна 22. При сжигании 4,4 г этого вещества образуется 13,2 г диоксида углерода и 7,2 г воды. Выведите молекулярную формулу органического соединения.

                  10 класс

                  Домашнее задание №1: «Теоретические основы органической химии.

                  Задачи на вывод формулы вещества»

                  ВАРИАНТ 3

                  Сравните по силе индуктивного эффекта следующие группы заместителей:

                    а) Nh3, N(Ch4)2, N(Ch4)3;

                    б) O, Oh3, OH;

                    в) гидроксильная группа, аминогруппа, меркаптогруппа;

                    г) Se, O, S;

                    д) =O, =N(Ch4).

                    Ответы объясните.

                    Напишите структурные формулы всех возможных изомерных алканов состава C9h30, в которых главная цепь состоит а) из семи атомов углерода, б) из шести атомов углерода, в) из пяти атомов углерода. Назовите эти вещества по международной номенклатуре. Укажите первичные, вторичные, третичные и четвертичные атомы углерода.

                      Газообразный углеводород имеет относительную плотность по кислороду 1,375, а массовые доли углерода и водорода в нем составляют соответственно 81,8% и 18,2%. Какое это соединение?

                        Относительная плотность паров органического соединения по хлору равна 2. При сжигании 2,84 г этого вещества образуется 4,48 л оксида углерода (IV) (н.у.) и 3,96 г воды. Выведите молекулярную формулу этого соединения.

                          10 класс

                          Домашнее задание №1: «Теоретические основы органической химии.

                          Задачи на вывод формулы вещества»

                          ВАРИАНТ 4

                          Укажите знак электронных эффектов для следующих заместителей: COOH, OH, CN, Br, O, CH=Ch3, SH, CH=O, S, аминогруппы, SO2OH, нитрогруппы, метоксигруппы, COO, Ch3, Ch3, B(Ch4)2, F, N=O. Ответы поясните.

                            Сколько изомеров может иметь соединение состава: а) C3H6Cl2; б) C4H9Cl? Назовите их.

                              Относительная плотность паров углеводорода по кислороду равна 2,25. Массовая доля углерода в нем равна 83,33%. Выведите молекулярную формулу этого углеводорода

                                При сгорании 31,4 г смеси двух алканов, различающихся по составу на два атома углерода, было получено 49,28 л (н.у.) углекислого газа. Определите состав исходной смеси.

                                  10 класс

                                  Домашнее задание №1: «Теоретические основы органической химии.

                                  Задачи на вывод формулы вещества»

                                  ВАРИАНТ 5

                                  Какие знаки индуктивных и мезомерных эффектов характерны для следующих заместителей: CN, NO2, Cl, OH, O, СH=O, SO2OH, COOH, COO, Ch3, Ch3, Br, OCh4, Nh3, NH, SH, SCh4, N(Ch4)2, F, I, C(O)Nh3, C(O)Cl, C(O)Ch4, S(O)-Ch4, O-NO2, CH=NH, O-C6H5, P(Ch4)2.

                                    Напишите структурные формулы следующих веществ:

                                      а) 2,2-диметил-3-этилгексан;

                                      б) 2,2,3-триметилгептен-1;

                                      в) 2-метил-4-пропил-3-этилоктан;

                                      г) 2,2,3-триметил-3-изопропилоктан;

                                      д) 2-метил-4-третбутилоктан.

                                      Плотность углеводорода при нормальных условиях равна 1,964 г/л. Массовая доля углерода в нем равна 81,82%. Выведите молекулярную формулу этого углеводорода.

                                        При сгорании 1,66 г смеси двух алканов, отличающихся по составу на один атом углерода, получили 5,06 г углекислого газа. Определить состав исходной смеси.

                                          10 класс

                                          Домашнее задание №1: «Теоретические основы органической химии.

                                          Задачи на вывод формулы вещества»

                                          ВАРИАНТ 6

                                          Какие из перечисленных ниже соединений и ионов относятся к кислотам, а какие к основаниям Льюиса: H, OH, BF3, h3O, этанол, Ag, F, SnCl4, Li, SO3, метиламин, цианид-анион, FeCl3, ZnCl2, Ch4, Ch4, h4O, NO, Ch4O, S2, Br, HF2, Hg2.

                                            Какой из перечисленных углеводородов с нормальной цепью (C4h20, C3H8, C7h26, C5h22, C8h28) будет иметь самую высокую температуру кипения?

                                              Относительная плотность паров циклоалкана по кислороду равна 3,5. Выведите молекулярную формулу циклоалкана.

                                                При сгорании 1,66 г смеси двух алканов, отличающихся по составу на один атом углерода, получили 5,06 г углекислого газа. Определить состав исходной смеси.

                                                   

                                                  10 класс

                                                  Домашнее задание №1: «Теоретические основы органической химии.

                                                  Задачи на вывод формулы вещества»

                                                  ВАРИАНТ 7

                                                  Напишите уравнения следующих кислотно-основных взаимодействий по Льюису:

                                                    Укажите кислоту и основание Льюиса в этих взаимодействиях. Ответ поясните.

                                                    Напишите структурные формулы всех возможных изомеров циклоалканов состава: а) C5h20; б) C6h22; Назовите эти вещества.

                                                      Плотность циклоалкана при нормальных условиях равна 1,875 г/л. Выведите молекулярную формулу циклоалкана.

                                                        Относительная плотность паров органического соединения по азоту равна 2. При сжигании 9,8 г этого соединения образуется 15,68 л углекислого газа (н. у) и 12,6 г воды. Выведите молекулярную формулу органического соединения.

                                                          10 класс

                                                          Домашнее задание №1: «Теоретические основы органической химии.

                                                          Задачи на вывод формулы вещества»

                                                          ВАРИАНТ 8

                                                          Определите тип разрыва (образования) химических связей для следующих реакций:

                                                            Напишите структурные формулы всех возможных изомеров циклоалканов состава: C7h24 в которых главная цепь состоит из:

                                                              а) четырех атомов углерода; б) пяти атомов углерода.

                                                              Назовите эти вещества.

                                                              Относительная плотность паров углеводорода по азоту равна 3. Массовая доля углерода в нем равна 85,71%. Выведите его молекулярную формулу.

                                                                Относительная плотность паров органического соединения по воздуху равна 4,83. При сжигании 17,5 г этого соединения получен углекислый газ, занимающий при н.у. объем 28 л, и вода массой 22,5 г. Выведите молекулярную формулу органического соединения.

                                                                  10 класс

                                                                  Домашнее задание №1: «Теоретические основы органической химии.

                                                                  Задачи на вывод формулы вещества»

                                                                  ВАРИАНТ 9

                                                                  Определите тип превращения для следующих реакций

                                                                    Напишите структурные формулы следующих веществ:

                                                                      а) 1,1-диметил-2-этил-3-изопропилциклопентан;

                                                                      б) 1,2-диэтил-3-изопропилциклогексан;

                                                                      в) 1,2-диметил-2,3-диэтил-3-пропилциклопентан.

                                                                      Относительная плотность паров углеводорода по водороду равна 35. Массовая доля водорода в нем равна 14,285%. Выведите молекулярную формулу углеводорода.

                                                                        7,5 г органического соединения при нормальных условиях занимает объем 4 л. При сжигании 2,1 г этого соединения образуется 6,6 г углекислого газа и 2,7 г воды. Выведите молекулярную формулу органического соединения.

                                                                          10 класс

                                                                          Домашнее задание №1: «Теоретические основы органической химии.

                                                                          Задачи на вывод формулы вещества»

                                                                          ВАРИАНТ 10

                                                                          Дайте определения следующим понятиям: нуклеофил, электрофил, радикал. Отнесите перечисленные ниже соединения и ионы к одному из вышеназванных реагентов: Ch4, Bh5, H, Nh4, Ch4, h3O, Cl, Ch3=Ch3, NO2, Cl, Br, Al3, Bh4, NO, Ch4-Ch3, H, ZnCl2, Nh3, Ch4-O, Nh3-Nh3, Li, OH, Ch4-S, Ch4-C(O)-O, SbF5, Ch4-Ch3-Li, Ch4-Mg-Cl, BF3, Na2S, SnCl4, H-F.

                                                                            Из предложенных формул выберите формулы алкенов:

                                                                              C3H6, C5h22, C7h24, C8h28, C10h30, C11h34, C12h34, C15h40, C16h44, C27H56.

                                                                              Относительная плотность паров алкена по водороду равна 42. Выведите молекулярную формулу алкена.

                                                                                относительная плотность паров органического вещества по кислороду равна 2,125. При сжигании 10,2 г этого вещества образовалось 16,8 л диоксида углерода (н.у.) и 10,8 г воды. Выведите молекулярную формулу этого вещества.

                                                                                  10 класс

                                                                                  Домашнее задание №1: «Теоретические основы органической химии.

                                                                                  Задачи на вывод формулы вещества»

                                                                                  ВАРИАНТ 11

                                                                                  Проведите классификацию следующих реакций по типу разрыва химических связей, типу реагента и типу превращения:

                                                                                    Напишите структурные формулы всех возможных изомерных алкенов состава: а) C4H8; б) C5h20; в) C6h22. Назовите эти вещества.

                                                                                      Плотность алкена при н. у. равна 2,5 г/л. Выведите молекулярную формулу алкена.

                                                                                        относительная плотность паров органического вещества по водороду равна 48. При сжигании 2,4 г этого вещества образовалось 3,92 л углекислого газа (н.у.) и 2,7 г воды. Выведите молекулярную формулу вещества.

                                                                                          10 класс

                                                                                          Домашнее задание №1: «Теоретические основы органической химии.

                                                                                          Задачи на вывод формулы вещества»

                                                                                          ВАРИАНТ 12

                                                                                          Определите конфигурации по R,S-номенклатуре следующих соединений, представленных в виде проекционных формул Фишера:

                                                                                            Напишите структурные формулы следующих веществ:

                                                                                              а) 2-метилпентен-1; б) 2,3,3-триметилбутен-1;

                                                                                              в) 4,5-диметилгексен-2; г) 2,3-диметилпентен-2.

                                                                                              Относительная плотность паров углеводорода по азоту равна 3,5. Массовая доля углерода в нем равна 85,71%. Выведите молекулярную формулу углеводорода.

                                                                                                27 г органического вещества при н.у. занимают объем 11,2 л. При сжигании 21,6 г этого вещества образуется 35,84 л углекислого газа (н.у.) и 21,6 г воды. Выведите молекулярную формулу органического вещества

                                                                                                  10 класс

                                                                                                  Домашнее задание №1: «Теоретические основы органической химии.

                                                                                                  Задачи на вывод формулы вещества»

                                                                                                  ВАРИАНТ 13

                                                                                                  Назовите следующие углеводородные радикалы:

                                                                                                    а) –СН3;

                                                                                                    б) –С2Н5;

                                                                                                    в) –СН(СН3)2;

                                                                                                    г) –СН(СН3)СН2СН3;

                                                                                                    д) –С(СН3)3;

                                                                                                    е) –(СН2)4СН(СН3)2;

                                                                                                    ж) –СН2СН(СН3)2.

                                                                                                    Напишите структурные формулы следующих веществ:

                                                                                                      а) цис-пентен-2; б) транс-пентен-2.

                                                                                                      Углеводород массой 12,5 г при н.у. занимает объем 5 л. Массовая доля водорода в нем равна 14,29%. Выведите молекулярную формулу углеводорода.

                                                                                                        Плотность паров органического соединения по неону равна 4,1. При сжигании 8,2 г этого вещества образуется 26,4 г оксида углерода (IV) и 9 г воды. Выведите молекулярную формулу органического вещества.

                                                                                                          10 класс

                                                                                                          Домашнее задание №1: «Теоретические основы органической химии.

                                                                                                          Задачи на вывод формулы вещества»

                                                                                                          ВАРИАНТ 14

                                                                                                          Укажите знак электронных эффектов для следующих заместителей: COOH, OH, CN, Br, O, CH=Ch3, SH, CH=O, S, аминогруппы, SO2OH, нитрогруппы, метоксигруппы, COO, Ch3, Ch3, B(Ch4)2, F, N=O. Ответы поясните.

                                                                                                            Напишите структурные формулы всех возможных изомерных алкадиенов состава: а) C5H8; б) C6h20. Назовите эти вещества.

                                                                                                              относительная плотность паров алкадиена по кислороду равна 3. Выведите молекулярную формулу алкадиена.

                                                                                                                Относительная плотность паров органического вещества по кислороду равна 2,44. При сгорании 3,9 г этого вещества образуется 13,2 г углекислого газа и 2,7 г воды. Выведите молекулярную формулу органического вещества.

                                                                                                                  10 класс

                                                                                                                  Домашнее задание №1: «Теоретические основы органической химии.

                                                                                                                  Задачи на вывод формулы вещества»

                                                                                                                  ВАРИАНТ 15

                                                                                                                  Назовите следующие соединения:

                                                                                                                    Напишите структурные формулы всех возможных изомерных алкадиенов состава C8h24, имеющих в главной цепи 6 атомов углерода. Назовите все вещества.

                                                                                                                      относительная плотность углеводорода по воздуху равна 2,345. Массовая доля углерода в нем равна 88,24%. Выведите молекулярную формулу углеводорода.

                                                                                                                        Относительная плотность паров органического соединения по гелию равна 15. При сжигании 18 г этого соединения образуется 26,4 г оксида углерода (IV) и 10,8 г воды. Выведите молекулярную формулу органического соединения.

                                                                                                                          10 класс

                                                                                                                          Домашнее задание №1: «Теоретические основы органической химии.

                                                                                                                          Задачи на вывод формулы вещества»

                                                                                                                          ВАРИАНТ 16

                                                                                                                          Назовите следующие соединения по заместительной номенклатуре ИЮПАК:


                                                                                                                             

                                                                                                                            Напишите структурные формулы следующих веществ:

                                                                                                                              а) 2,4-диметилпентадиен-1,4; б) 2,3-диметил-4-этилгептадиен-1,3;

                                                                                                                              в) 2-метил-3-этилпентадиен-1,3; г) 2-метил-3,4-диэтилгептадиен-2,4.

                                                                                                                              1 г алкина при н.у. занимает объем 0,86 л. Выведите молекулярную формулу алкина.

                                                                                                                                Относительная плотность по гелию паров углеводорода ряда бензола равна 23. Выведите его молекулярную формулу.

                                                                                                                                  При полном сгорании 5,4 г некоторого органического соединения, относительная молекулярная масса которого равна 180, образовалось 4,032 л (н.у.) углекислого газа и 3,24 г воды. Определите молекулярную формулу этого соединения и предложите возможные структуры.

                                                                                                                                    10 класс

                                                                                                                                    Домашнее задание №1: «Теоретические основы органической химии.

                                                                                                                                    Задачи на вывод формулы вещества»

                                                                                                                                    ВАРИАНТ 17

                                                                                                                                    Какие из перечисленных ниже соединений и ионов относятся к кислотам, а какие к основаниям Льюиса: H, OH, BF3, h3O, этанол, Ag, F, SnCl4, Li, SO3, метиламин, цианид-анион, FeCl3, ZnCl2, Ch4, Ch4, h4O, NO, Ch4O, S2, Br, HF2, Hg2.

                                                                                                                                      Какие из перечисленных ниже углеводородов являются гомологами?

                                                                                                                                        Пропан, бутен-1, метилпропен, метилбутан, пентен-2,

                                                                                                                                        диметилпропан, 2-метилнонан, 2-метилбутен-2, н-гептан, гептен-1, октен-1, 2,2-диметилгексан, 2-метилгексен-2, гексен-2,

                                                                                                                                        бутен-2, 2-метилбутен-1.

                                                                                                                                        Относительная плотность паров алкина по воздуху равна 2,83. Выведите молекулярную формулу алкина.

                                                                                                                                          Три разных органических соединения имеют следующий состав: углерод – 40%, водород – 6,67%, кислород – 53,33%. Относительная молекулярная масса первого соединения равна 60, второго — 120, а третьего – 180. Определите их молекулярные формулы.

                                                                                                                                            10 класс

                                                                                                                                            Домашнее задание №1: «Теоретические основы органической химии.

                                                                                                                                            Задачи на вывод формулы вещества»

                                                                                                                                            ВАРИАНТ 18

                                                                                                                                            Какие знаки индуктивных и мезомерных эффектов характерны для следующих заместителей: CN, NO2, Cl, OH, O, СH=O, SO2OH, COOH, COO, Ch3, Ch3, Br, OCh4, Nh3, NH, SH, SCh4, N(Ch4)2, F, I, C(O)Nh3, C(O)Cl, C(O)Ch4, S(O)-Ch4, O-NO2, CH=NH, O-C6H5, P(Ch4)2.

                                                                                                                                              Напишите структурные формулы всех возможных изомерных алкинов состава: а) C5H8; б) C6h20.Назовите эти вещества по международной номенклатуре.

                                                                                                                                                Относительная плотность паров углеводорода по водороду равна 41. Массовая доля водорода в нем равна 12,2%. Выведите молекулярную формулу углеводорода.

                                                                                                                                                  Три разных органических соединения имеют следующий состав: углерод – 40%, водород – 6,67%, кислород – 53,33%. Относительная молекулярная масса первого соединения равна 60, второго — 120, а третьего – 180. Определите их молекулярные формулы.

                                                                                                                                                    10 класс

                                                                                                                                                    Домашнее задание №1: «Теоретические основы органической химии.

                                                                                                                                                    Задачи на вывод формулы вещества»

                                                                                                                                                    ВАРИАНТ 19

                                                                                                                                                    Дайте определения следующим понятиям: размер атома, электроотрицательность. Расположите следующие группы атомов в порядке возрастания их размера, их электроотрицательности:

                                                                                                                                                      a) O, N, C, F, Li, Na;

                                                                                                                                                      б) S, P, N, Cl, Al, Si;

                                                                                                                                                      в) Cl, Br, F, Se, As, O;

                                                                                                                                                      г) С, Cl, Li, Be, N.


                                                                                                                                                       

                                                                                                                                                      Какой из вышеприведенных атомов имеет наибольшую, а какой наименьшую электроотрицатальность?


                                                                                                                                                         

                                                                                                                                                        Напишите структурные формулы всех возможных изомерных алкинов состава C8h24, имеющих в главной цепи 6 атомов углерода. Назовите эти вещества по международной номенклатуре.

                                                                                                                                                          Относительная плотность паров углеводорода по кислороду равна 2,125. Массовая доля углерода в нем равна 88,24%. Выведите молекулярную формулу углеводорода.

                                                                                                                                                            10 класс

                                                                                                                                                            Домашнее задание №1: «Теоретические основы органической химии.

                                                                                                                                                            Задачи на вывод формулы вещества»

                                                                                                                                                            ВАРИАНТ 20

                                                                                                                                                            Укажите знак электронного эффекта для следующих заместителей: COOH, COO, C2H5, Ch4C=O, Br, Li, O, CN, Nh4, SH, HgC2H5, Si(Ch4)3, CH=O, =O, C(Ch4)3, S, OH. Ответы поясните.

                                                                                                                                                              Напишите структурные формулы всех гомологов бензола с молекулярной формулой а) C7H8; б) C8h20; в) C9h22. Назовите эти вещества по международной номенклатуре.

                                                                                                                                                                пары арена имеют относительную плотность по воздуху 4,14. Выведите его молекулярную формулу.

                                                                                                                                                                  Относительная плотность паров углеводорода по углекислому газу равна 2,41. Массовая доля углерода в нем равна 90,6%. Выведите молекулярную формулу углеводорода.

                                                                                                                                                                    10 класс

                                                                                                                                                                    Домашнее задание №1: «Теоретические основы органической химии.

                                                                                                                                                                    Задачи на вывод формулы вещества»

                                                                                                                                                                    ВАРИАНТ 21

                                                                                                                                                                    Напишите структурные формулы всех изомерных спиртов с молекулярной формулой: а) C4h20O; б) C5h22O. Назовите эти вещества по международной номенклатуре.

                                                                                                                                                                      Относительная плотность паров предельного одноатомного спирта по водороду равна 37. Выведите молекулярную формулу спирта.

                                                                                                                                                                        Относительная плотность паров органического соединения по кислороду равна 2,75. Массовая доля углерода в этом веществе равна 68,18%, массовая доля водорода равна 13,64%, остальное — кислород. Выведите молекулярную формулу этого вещества.

                                                                                                                                                                          В результате сгорания 0,828 г органического соединения образовалось 1,584 г углекислого газа и 0,972 г воды. Относительная плотность паров этого соединения по метану 2,875. Установите формулу этого соединения

                                                                                                                                                                            10 класс

                                                                                                                                                                            Домашнее задание №1: «Теоретические основы органической химии.

                                                                                                                                                                            Задачи на вывод формулы вещества»

                                                                                                                                                                            ВАРИАНТ 22

                                                                                                                                                                            Какие из перечисленных ниже соединений и ионов относятся к кислотам, а какие к основаниям Льюиса: H, OH, BF3, h3O, этанол, Ag, F, SnCl4, Li, SO3, метиламин, цианид-анион, FeCl3, ZnCl2, Ch4, Ch4, h4O, NO, Ch4O, S2, Br, HF2, Hg2.

                                                                                                                                                                              Напишите структурные формулы всех изомерных карбоновых кислот с молекулярной формулой: а) C3H6O2; б) C4H8O2; в) C6h20O2. Назовите эти вещества.

                                                                                                                                                                                Относительная плотность паров органического соединения по кислороду равна 4,5. Массовая доля углерода в этом веществе равна 66,67%, массовая доля водорода равна 11,11%, массовая доля кислорода равна 22,22%. Выведите молекулярную формулу органического соединения.

                                                                                                                                                                                  Относительная плотность паров органического соединения по сернистому газу равна 2. При сжигании 19,2 г этого вещества образуется 52,8 г углекислого газа (н. у.) и 21,6 г воды. Выведите молекулярную формулу органического соединения.

                                                                                                                                                                                    10 класс

                                                                                                                                                                                    Домашнее задание №1: «Теоретические основы органической химии.

                                                                                                                                                                                    Задачи на вывод формулы вещества»

                                                                                                                                                                                    ВАРИАНТ 23

                                                                                                                                                                                    Сравните по силе индуктивного эффекта следующие группы заместителей:

                                                                                                                                                                                      а) Nh3, N(Ch4)2, N(Ch4)3;

                                                                                                                                                                                      б) O, Oh3, OH;

                                                                                                                                                                                      в) гидроксильная группа, аминогруппа, меркаптогруппа;

                                                                                                                                                                                      г) Se, O, S; ж) =O, =N(Ch4).

                                                                                                                                                                                      Ответы объясните.

                                                                                                                                                                                      Напишите структурные формулы следующих карбоновых кислот:

                                                                                                                                                                                        а) 2,2,4-триметил-3-этилгексановая кислота;

                                                                                                                                                                                        б) 3,4,5,5-тетраметил-3,4-диэтилгептановая кислота;

                                                                                                                                                                                        в) 2,2,3-триметил-3-этилпентановая кислота;

                                                                                                                                                                                        г) 2-метил-4-изопропил-3-этилгексановая кислота.

                                                                                                                                                                                        д) 2,4-диметил-3-гексеновая кислота.

                                                                                                                                                                                        Для сгорания 6 моль вещества потребовалось 13,5 моль кислорода и образовалось 6 моль углекислого газа, 3 моль азота и 15 моль воды. Установите молекулярную формулу вещества.

                                                                                                                                                                                          Плотность паров органического соединения по неону равна 4,1. При сжигании 8,2 г этого вещества образуется 26,4 г оксида углерода (IV) и 9 г воды. Выведите молекулярную формулу органического вещества

                                                                                                                                                                                            10 класс

                                                                                                                                                                                            Домашнее задание №1: «Теоретические основы органической химии.

                                                                                                                                                                                            Задачи на вывод формулы вещества»

                                                                                                                                                                                            ВАРИАНТ 24

                                                                                                                                                                                            Напишите структурные формулы изомерных аминов с общей молекулярной формулой: а) CH5N; б) CH6N2; в) C4h21N; г) C5h23N; д) C3H7N; е) C4H9N. Назовите эти вещества.

                                                                                                                                                                                              Даны вещества: циклобутан, ацетилен, гептан, бутен-1, бутин-2, циклогексан, 2,2,3-триметилбутан, бутадиен. Вы­берите среди них а) три пары изомеров, б) две пары гомологов.

                                                                                                                                                                                                Массовая доля углерода в моноамине равна 63,16%, массовая доля азота равна 24,56%. Выведите молекулярную формулу амина.

                                                                                                                                                                                                  Относительная плотность паров предельного амина по кислороду равна 1,844. Выведите молекулярную формулу амина.

                                                                                                                                                                                                    10 класс

                                                                                                                                                                                                    Домашнее задание №1: «Теоретические основы органической химии.

                                                                                                                                                                                                    Задачи на вывод формулы вещества»

                                                                                                                                                                                                    ВАРИАНТ 25

                                                                                                                                                                                                    Дайте определения следующим понятиям: нуклеофил, электрофил, радикал. Отнесите перечисленные ниже соединения и ионы к одному из вышеназванных реагентов: Ch4, Bh5, H, Nh4, Ch4, h3O, Cl, Ch3=Ch3, NO2, Cl, Br, Al3, Bh4, NO, Ch4-Ch3, H, ZnCl2, Nh3, Ch4-O, Nh3-Nh3, Li, OH, Ch4-S, Ch4-C(O)-O, SbF5, Ch4-Ch3-Li, Ch4-Mg-Cl, BF3, Na2S, SnCl4, H-F.

                                                                                                                                                                                                      Напишите структурные формулы всех изомерных веществ с молекулярной формулой: а) C3H7O2N; б) C4H9O2N.

                                                                                                                                                                                                        Относительная плотность паров алкина по воздуху равна 2,83. Выведите молекулярную формулу алкина.

                                                                                                                                                                                                          При полном сгорании 5,4 г некоторого органического соединения, относительная молекулярная масса которого равна 180, образовалось 4,032 л (н.у.) углекислого газа и 3,24 г воды. Определите молекулярную формулу этого соединения и предложите возможные структуры.

                                                                                                                                                                                                            Алгоритм решения задач по определению молекулярной формулы вещества

                                                                                                                                                                                                            по его относительной плотности и массовой доле элементов в соединении

                                                                                                                                                                                                            Mr = 2∙; или Mr= 29∙Dвозд; или Mr = 32∙; или , где Vm = 22,4 л/моль

                                                                                                                                                                                                            ЗАДАЧИ.

                                                                                                                                                                                                          1. Выведите формулу вещества, содержащего 82,75% углерода и 17,25% водорода. Относительная плотность паров этого вещества по воздуху равна 2.

                                                                                                                                                                                                          2. Выведите формулу вещества, содержащего 81,8% углерода и 18,2% водорода, если относительная плотность по водороду равна 22.

                                                                                                                                                                                                          3. Выведите молекулярную формулу углеводорода, содержащего 85,71% углерода и 14,29% водорода. Относительная плотность паров этого вещества по водороду равна 21.

                                                                                                                                                                                                          4. Определите молекулярную формулу углеводорода с массовой долей углерода 85,7%. Относительная плотность данного вещества по водороду равна 14.

                                                                                                                                                                                                          5. Вычислите относительную молекулярную массу и его формулу, если массовая доля в нем углерода равна 92,3%, а водорода – 7,7%. Относительная плотность данного вещества по водороду равна 13.

                                                                                                                                                                                                          6. Определите молекулярную формулу углеводорода, если массовая доля углерода равна 80%, а водорода – 20%. Относительная плотность данного вещества по водороду равна 15.

                                                                                                                                                                                                          7. Определите молекулярную формулу органического соединения, если массовая доля в нем углерода равна 37,5%, кислорода – 50%. водорода – 12,5%. Относительная плотность данного соединения по водороду равна 16.

                                                                                                                                                                                                          8. Выведите молекулярную формулу фторпроизводного предельного углеводорода с массовой долей фтора, равной 0,73, углерода – 0,23 и водорода – 0,04. Относительная молекулярная масса этого соединения равна 52.

                                                                                                                                                                                                          9. Определите молекулярную формулу углеводорода, если массовая доля углерода в нем равна 82,8%, а плотность этого вещества составляет 2,59 г/л.

                                                                                                                                                                                                          10. Какова молекулярная формулу вещества, в котором массовая доля углерода равна 0,52, кислорода – 0,35, водорода – 0,13? Относительная плотность вещества по водороду равна 23.

                                                                                                                                                                                                          11. Органическое вещество, массовая доля углерода в котором равна 65,75%, водорода – 15,06%, азота – 19,18%, имеет относительную плотность паров этого вещества по воздуху 2,52. Выведите молекулярную формулу вещества.

                                                                                                                                                                                                          12. Органическое вещество, в котором массовая доля углерода равна 53,3%, водорода – 15,6%, азота – 31,1%, имеет относительную плотность по воздуху 1,551. Выведите молекулярную формулу вещества.

                                                                                                                                                                                                          13. Основные величины для решения задач

                                                                                                                                                                                                            Mr

                                                                                                                                                                                                            относительная молекулярная масс вещества

                                                                                                                                                                                                            относительная плотность паров вещества по водороду

                                                                                                                                                                                                            относительная плотность паров вещества по кислороду

                                                                                                                                                                                                            Dвозд

                                                                                                                                                                                                            относительная плотность паров вещества по воздуху

                                                                                                                                                                                                            молярный объем (постоянная величина 22,4 л/моль)

                                                                                                                                                                                                            плотность вещества

                                                                                                                                                                                                            ω(Э)

                                                                                                                                                                                                            массовая доля элемента

                                                                                                                                                                                                            относительная атомная масса элемента

                                                                                                                                                                                                            количество атомов химического элемента в сложном веществе

                                                                                                                                                                                                            Задачи по химии на вывод формул,желательно с решением — Учеба и наука

                                                                                                                                                                                                            1) Запишем уравнение  реакции в общем виде 
                                                                                                                                                                                                            Сnh3nO2 + NaHCO3 = Cnh3n-1O2Na + CO2 + h3O 
                                                                                                                                                                                                            Из уравнение следует, что из 1 моля кислоты получается 1 моль газа СО2 
                                                                                                                                                                                                            найдём  число молей газа:

                                                                                                                                                                                                            ν(СО2) = 5,6/22,4 = 0,25 моль. Следовательно, и кислоты было 0,25 моль. Т. е. молярная масса кислоты 22/0,25 = 88 г/моль 
                                                                                                                                                                                                            Теперь составляем уравнение из формулы кислоты, зная атомные массы углерода, водорода и кислорода: 
                                                                                                                                                                                                            12n +2n + 32 = 88

                                                                                                                                                                                                             n = 4

                                                                                                                                                                                                             формула кислоты С4Н10О2 или С3Н7СООН

                                                                                                                                                                                                            2) Найдём молярную масса вещества = 1,8125 * 32 = 58 г/моль, отсюда количество = 1,74/58 = 0,03 моля.
                                                                                                                                                                                                            Так как количества углекислого газа и воды равны их суммарная масса будет равна х(Mr(CO2) + Mr(h3O)) = 5,58 г, отсюда х = 0,09 моля.
                                                                                                                                                                                                            Узнаем сколько углерода в молекуле 0,09/0,03 = 3 атома. Узнаем сколько водорода в молекуле = 0,09*2/0,03 = 6 атомов. Полученный C3H6 имеет молярную массу 42, недостающая масса — это кислород (1 атом).

                                                                                                                                                                                                            C3H6O

                                                                                                                                                                                                            3) в-во+ О2→СО2+Н2О

                                                                                                                                                                                                            в состав сгораемого вещества обязательно входят углерод и водород, но возможно наличие кислорода. Поэтому, необходимо сделать проверку на кислород:

                                                                                                                                                                                                            ν(СО2)=m/M=35,2 г/44 г/моль=0,8 моль;  ν©=0,8 моль; m©=9,6г

                                                                                                                                                                                                            ν(Н2О)=m/M=14,4 г/18 г/моль=0,8 моль; ν(Н)=0,8 моль; m(Н)=1,6 г

                                                                                                                                                                                                            m©+m(H)= 9,6г + 1,6 г=11,2 г

                                                                                                                                                                                                            а вещества сгорело 24 г, следовательно, имеется кислород.

                                                                                                                                                                                                            m(О)=24г – 11,2 г=12,8 г

                                                                                                                                                                                                            ν(О)=12,8 г/16 г/моль=0,8моль

                                                                                                                                                                                                            СхНyОz х:y:z= ν©: ν(Н): ν(О)=0,8:1,6:0,8

                                                                                                                                                                                                            Находим среди этих цифр самое маленькое и все делим на него, получая: х:y:z= 1:2:1

                                                                                                                                                                                                            СН2О-простейшая формула.

                                                                                                                                                                                                            D(водороду)=M(в-ва)/М(водорода)

                                                                                                                                                                                                            М (в-ва)= D(водороду)×М(водороду)=30 ×2 г/моль=60 г/моль

                                                                                                                                                                                                            M (СН2О)=12+1*2+16=30 г/моль,

                                                                                                                                                                                                            что не соответствует действительности, следовательно надо увеличить в 2 раза

                                                                                                                                                                                                            С2Н4О2   или СН3СООН – уксусная кислота

                                                                                                                                                                                                            не забывайте отмечать эксперта выбором лучшего ответа

                                                                                                                                                                                                            Задачи на вывод молекулярной формулы. I тип Относительная плотность паров алкана по водороду равна 50. Выведите молекулярную формулу алкана

                                                                                                                                                                                                            Задачи на вывод молекулярной формулы. I тип

                                                                                                                                                                                                            1. Относительная плотность паров алкана по водороду равна 50. Выведите молекулярную формулу алкана.

                                                                                                                                                                                                            2. Относительная плотность паров алкина по кислороду равна 2,125. Выведите молекулярную формулу алкина.

                                                                                                                                                                                                            3. Плотность алкадиена при нормальных условиях равна 2,41г/л. Выведите молекулярную формулу алкадиена.

                                                                                                                                                                                                            4. Относительная плотность паров алкадиена по диоксиду углерода равна 2,5. Выведите молекулярную формулу алкадиена.

                                                                                                                                                                                                            5. Относительная плотность паров предельной одноосновной карбоновой кислоты по воздуху равна 2,552. Выведите молекулярную формулу карбоновой кислоты.

                                                                                                                                                                                                            6. Относительная плотность паров предельной двухосновной карбоновой кислоты по аммиаку равна 5,29. Выведите молекулярную формулу карбоновой кислоты.

                                                                                                                                                                                                            7. Относительная плотность паров предельного двухатомного спирта по угарному газу равна 2,214. Выведите молекулярную формулу спирта.

                                                                                                                                                                                                            8. Относительная плотность паров циклоалкана по азоту равна 5. Выведите молекулярную формулу циклоалкана.

                                                                                                                                                                                                            9. Гомолог бензола имеет относительную плотность паров по воздуху 4,62. Выведите его молекулярную формулу.

                                                                                                                                                                                                            10. Относительная плотность паров предельного трёхатомного спирта по воздуху равна 4,62. Выведите молекулярную формулу спирта.

                                                                                                                                                                                                            Задачи на вывод молекулярной формулы. II тип


                                                                                                                                                                                                            1. Относительная плотность паров углеводорода по кислороду равна 3. Массовая доля водорода в нём равна 12,5%. Выведите молекулярную формулу углеводорода.

                                                                                                                                                                                                            2. Относительная плотность паров органического соединения по водороду равна 50. Массовая доля углерода в веществе равна 72%, массовая доля водорода равна 12%, массовая доля кислорода равна 16%. Выведите молекулярную формулу органического соединения.

                                                                                                                                                                                                            3. Относительная плотность паров углеводорода по азоту равна 3. Массовые доли углерода и водорода в нём равны соответственно 85,71% и 14,29%. Выведите молекулярную формулу углеводорода.

                                                                                                                                                                                                            4. Относительная плотность паров органического соединения по воздуху равна 2,552. Массовая доля углерода в веществе равна 64,86%, массовая доля водорода равна 13,51%, массовая доля кислорода равна 21,63%. Выведите молекулярную формулу этого вещества.

                                                                                                                                                                                                            5. Массовая доля углерода в циклоалкане составляет 85,71%. Относительная плотность его паров по воздуху равна 1,931. Найдите молекулярную формулу циклоалкана.

                                                                                                                                                                                                            6. Относительная плотность паров органического соединения по водороду равна 55. Массовая доля углерода в веществе равна 65,45%, массовая доля водорода равна 5,45%, массовая доля кислорода равна 29,1%. Выведите молекулярную формулу органического соединения.

                                                                                                                                                                                                            7. Относительная плотность паров углеводорода по водороду равна 35. Массовые доли углерода и водорода в нём равны соответственно 85,715% и 14,285%. Выведите молекулярную формулу углеводорода.

                                                                                                                                                                                                            8. Плотность углеводорода при нормальных условиях равна 2,59г/л. Массовая доля углерода в нём равна 82,76%. Выведите молекулярную формулу этого соединения.

                                                                                                                                                                                                            9. Относительная плотность паров углеводорода по водороду равна 55. Массовая доля углерода в нём равна 87,27%. Выведите молекулярную формулу углеводорода.

                                                                                                                                                                                                            10. Углеводород массой 25г при нормальных условиях занимает объём 10л. Массовая доля углерода в нём равна 85,71%. Выведите молекулярную формулу углеводорода.

                                                                                                                                                                                                            Задачи на вывод молекулярной формулы. III тип


                                                                                                                                                                                                            1. Относительная плотность паров органического вещества по водороду равна 45. При сжигании 5,4 г этого вещества образовалось 5,376 л углекислого газа (н.у.) и 5,4 г воды. Выведите молекулярную формулу органического вещества.

                                                                                                                                                                                                            2. Относительная плотность органического соединения по кислороду равна 1,375. При сжигании 4,4 г этого вещества образуется 13,2 г диоксида углерода и 7,2 г воды. Выведите молекулярную формулу этого органического соединения.

                                                                                                                                                                                                            3. При сжигании углеводорода объёмом 2,24 л получили 13,2 г оксида углерода(IV) и 7,2 г воды. Относительная плотность углеводорода по водороду равна 22. Найдите молекулярную формулу углеводорода.

                                                                                                                                                                                                            4. Относительная плотность паров органического вещества по водороду равна 30. При сжигании 24 г этого вещества образовалось 35,2 г диоксида углерода и 14,4 г воды. Выведите молекулярную формулу органического соединения.

                                                                                                                                                                                                            5. Относительная плотность паров органического вещества по воздуху равна 3,31. При сжигании 24 г этого вещества образовалось 39,2 л углекислого газа (н.у. ) и 27 г воды. Выведите молекулярную формулу органического вещества.

                                                                                                                                                                                                            6. 13,5г органического вещества при нормальных условиях занимают объём 5,6л. При сжигании 10,8 г этого вещества образуется 17,92 л углекислого газа (н.у.) и 10,8 г воды. Выведите молекулярную формулу органического вещества.

                                                                                                                                                                                                            7. Относительная плотность паров органического соединения по водороду равна 62. При сжигании 24,8 г этого вещества образуется 31,36 л углекислого газа (н.у.) и 14,4 г воды. Выведите молекулярную формулу органического соединения.

                                                                                                                                                                                                            8. Плотность паров органического соединения по гелию равна 20,5. При сжигании 4,1 г этого вещества образуется 13,2 г диоксида углерода и 4,5 г воды. Выведите молекулярную формулу органического вещества.

                                                                                                                                                                                                            9. При горении 1 моль газообразного алкана (при н.у.) образовалось 22,4 л оксида углерода(IV) и 36 г воды. Найдите молекулярную формулу алкана.

                                                                                                                                                                                                            10. Относительная плотность паров органического вещества по воздуху равна 2,69. При сгорании 19,5 г этого вещества образовалось 66 г углекислого газа и 13,5 г воды. Выведите молекулярную формулу органического вещества.

                                                                                                                                                                                                            Дальнейшая часть истории: Расчет плотности пара

                                                                                                                                                                                                            На протяжении многих лет американцы с удовольствием смотрели выпуски новостей и современную информацию, которые передавал по радио Пол Харви. В частности, одна из частей его шоу, раскрывающая очень интересные факты, — это «Остальная часть истории». Харви обычно сообщает слушателям гораздо больше негласной информации, чем большинство людей слышали по определенной теме. Обычно, когда рассказ завершен, слушатель лучше понимает, почему все так, как есть, — если хотите, остальную часть истории.

                                                                                                                                                                                                            Эта же концепция во многих отношениях применима и к реагированию на опасные материалы. Одна из тем, где остальная часть сюжетной информации может реально повысить безопасность спасателей, — это плотность пара. Большинство респондентов были обучены некоторым вещам о плотности пара, но это гораздо больше, чем то, что обычно включается в большинство тренировок по хазмату. На самом деле плотность пара обычно изображается как общее понятие, но в интересах безопасности необходимо более полное представление.Следовательно, точные расчеты, касающиеся концепции плотности пара, позволят персоналу, осуществляющему реагирование, безопасно реагировать на аварийные ситуации с опасными материалами и проводить более эффективные оценки рисков.

                                                                                                                                                                                                            Плотность пара определяется как относительный вес газа или пара по сравнению с воздухом, который имеет произвольное значение, равное единице. Если газ имеет плотность пара менее единицы, он обычно поднимается в воздух. Если плотность пара больше единицы, газ обычно тонет в воздухе.

                                                                                                                                                                                                            Эта концепция важна для лиц, осуществляющих реагирование, потому что она указывает, где обычно можно ожидать расположения газа или паров при выбросах опасных веществ. Соответственно, лица, ответственные за реагирование, могут лучше выбирать места размещения и установки оборудования, а также зоны изоляции опасности для инцидента. Кроме того, знание того, где может быть обнаружен газ, может лучше определить стратегию мониторинга при каждом инциденте.

                                                                                                                                                                                                            Единственная проблема заключается в том, что концепция плотности пара, которую также называют «удельным весом пара», предлагает лишь неопределенное значение того, куда может перемещаться газ или пар.Другими словами, руководства по реагированию, такие как Руководство по реагированию на чрезвычайные ситуации (ERG) Министерства транспорта, в которых говорится, что «пары могут быть тяжелее воздуха и обнаруживаться в низинах», не указывают, насколько тяжелыми будут пары. Если быть более точным, то можно рассчитать плотность пара.

                                                                                                                                                                                                            Плотность воздуха

                                                                                                                                                                                                            Воздух представляет собой сложную смесь нескольких газов, среди которых преобладают азот и кислород. Весовой состав воздуха на уровне моря:

                                                                                                                                                                                                            • Азот — 75.53%
                                                                                                                                                                                                            • Кислород — 23,16%
                                                                                                                                                                                                            • Прочие — 1,31% (аргон, диоксид углерода, неон, гелий, метан, криптон, закись азота, водород, ксенон, озон)

                                                                                                                                                                                                            По объему воздух состоит из следующих газов:

                                                                                                                                                                                                            • Азот — 78,00%
                                                                                                                                                                                                            • Кислород — 20,95%
                                                                                                                                                                                                            • Аргон — 0,93%
                                                                                                                                                                                                            • CO2 — 0,033%
                                                                                                                                                                                                            • Прочие —

                                                                                                                                                                                                            Кроме того, воздух имеет молекулярную массу 29 атомных единиц массы (а. е.м.) на уровне моря.По сути, это вес пробы воздуха, который можно использовать для сравнения с другими газами и парами.

                                                                                                                                                                                                            Плотность газов

                                                                                                                                                                                                            Учитывая идентичность материала, его молекулярный вес можно рассчитать по его химической формуле и в сочетании с периодической таблицей элементов. Все атомы имеют массу, а вес — притяжение массы за счет силы тяжести. В наших целях мы называем массу соединения его весом. Определив химическую формулу соединения и сложив массу всех его атомов, можно рассчитать молекулярную массу.Например, молекулярная масса безводного аммиака равна 17, потому что формула Nh4, где один атом азота (N) равен 14 а.е.м., а три атома водорода (H) — 3 а.е.м.

                                                                                                                                                                                                            Расчет плотности пара

                                                                                                                                                                                                            Чтобы вычислить плотность пара соединения, просто разделите молекулярную массу соединения на молекулярную массу воздуха. Это даст числовое значение, которое можно сравнить со значением единицы воздуха.

                                                                                                                                                                                                            Например, молекулярная масса водорода составляет 2 а. е.м. (газообразный водород двухатомный), а молекулярная масса воздуха — 29 (28.9 если быть точным). Частное 2/29 равно 0,068. Поскольку этот ответ меньше единицы, водород поднимется в воздух (помните Гинденбург?). И наоборот, такой продукт, как гексан, выделяет пары, которые тонут в воздухе. Расчет веса гексана основан на его химической формуле C6h24, которая дает молекулярную массу 84 а.е.м. Частное 84/29 составляет 2,9. Таким образом, пары гексана в 2,9 раза тяжелее воздуха.

                                                                                                                                                                                                            Мнемоника плотности пара

                                                                                                                                                                                                            Чтобы помочь запомнить, какие газы легче воздуха, были придуманы некоторые мнемоники или акронимы.На рубеже веков пожарный из Нью-Йорка разработал хорошо известную мнемонику для определения плотности пара. Для обучения своих товарищей-пожарных он использовал термин «HA HA MICE», чтобы помнить о газах легче воздуха. Буквы обозначают;

                                                                                                                                                                                                            H — Водород
                                                                                                                                                                                                            A — Аммиак
                                                                                                                                                                                                            H — Гелий
                                                                                                                                                                                                            A — Ацетилен

                                                                                                                                                                                                            M — Метан
                                                                                                                                                                                                            I — Светящиеся газы (старый термин для природного газа)
                                                                                                                                                                                                            C — Окись углерода
                                                                                                                                                                                                            E — Этилен

                                                                                                                                                                                                            Этот акроним использовался в течение многих лет, но сегодня мы знаем, что существует более восьми газов, которые легче (или того же веса), что и воздух. Чтобы запомнить 13 газов, которые легче воздуха, в качестве мнемоники можно использовать новую аббревиатуру. Термин «4H MEDIC ANNA» обозначает газы легче воздуха, и они есть;

                                                                                                                                                                                                            Газ
                                                                                                                                                                                                            Молекулярная формула
                                                                                                                                                                                                            АМУ
                                                                                                                                                                                                            Плотность пара
                                                                                                                                                                                                            H — Водород
                                                                                                                                                                                                            h3
                                                                                                                                                                                                            2
                                                                                                                                                                                                            .07
                                                                                                                                                                                                            H — Гелий
                                                                                                                                                                                                            Он
                                                                                                                                                                                                            4
                                                                                                                                                                                                            .14 ​​
                                                                                                                                                                                                            H — Цианистый водород
                                                                                                                                                                                                            HCN
                                                                                                                                                                                                            29
                                                                                                                                                                                                            1.0
                                                                                                                                                                                                            H — Фтористый водород
                                                                                                                                                                                                            ВЧ
                                                                                                                                                                                                            10
                                                                                                                                                                                                            . 34
                                                                                                                                                                                                            M — Метан
                                                                                                                                                                                                            Ч5
                                                                                                                                                                                                            16
                                                                                                                                                                                                            .55
                                                                                                                                                                                                            E — Этилен
                                                                                                                                                                                                            C2h5
                                                                                                                                                                                                            28
                                                                                                                                                                                                            0,96
                                                                                                                                                                                                            D — Диборан
                                                                                                                                                                                                            B2H6
                                                                                                                                                                                                            27.7
                                                                                                                                                                                                            0,96
                                                                                                                                                                                                            I — Световые газы
                                                                                                                                                                                                            Ch5 / C2H6
                                                                                                                                                                                                            17,4
                                                                                                                                                                                                            .6
                                                                                                                                                                                                            C — Окись углерода
                                                                                                                                                                                                            CO
                                                                                                                                                                                                            28
                                                                                                                                                                                                            0,96
                                                                                                                                                                                                            A — Ацетилен
                                                                                                                                                                                                            C2h3
                                                                                                                                                                                                            26
                                                                                                                                                                                                            .9
                                                                                                                                                                                                            N — Неон
                                                                                                                                                                                                            Ne
                                                                                                                                                                                                            10
                                                                                                                                                                                                            .34
                                                                                                                                                                                                            N — Азот
                                                                                                                                                                                                            N2
                                                                                                                                                                                                            28
                                                                                                                                                                                                            0,96
                                                                                                                                                                                                            A — Аммиак
                                                                                                                                                                                                            Кh4
                                                                                                                                                                                                            17
                                                                                                                                                                                                            . 59
                                                                                                                                                                                                            Примечание; Осветляющие газы — это «природный газ», который представляет собой смесь примерно 90% метана и 10% этана.

                                                                                                                                                                                                            Если вы помните эту мнемонику для газов легче воздуха, все остальное тяжелее, включая пары легковоспламеняющихся жидкостей. В целом, чем тяжелее пар, тем меньше он будет накапливаться при выпуске.

                                                                                                                                                                                                            Меры предосторожности и применение

                                                                                                                                                                                                            Плотность пара — это просто общая концепция, позволяющая понять, где пары могут быть обнаружены при выбросе.Однако этот физический параметр не является абсолютным и на него могут влиять:

                                                                                                                                                                                                            • воздушные потоки — смешивают все газы и пары, несмотря на различную плотность пара.
                                                                                                                                                                                                            • температура — при которой газы и пары могут подниматься или опускаться.
                                                                                                                                                                                                            • Кроме того, материал, выходящий из контейнера под давлением, может изменить ожидаемую плотность пара.
                                                                                                                                                                                                            • влажность — которая может поглощаться газами или парами, что снижает их плавучесть.
                                                                                                                                                                                                            • точка росы — которая позволяет водяному пару в воздухе подниматься, что может повлиять на поведение паров.
                                                                                                                                                                                                            • аэрозоли — наличие мелких капель в парах может сделать облако тяжелым.

                                                                                                                                                                                                            Еще одна мера предосторожности, связанная с концепцией плотности пара, — это допущение, что отношения между газом или паром и воздухом являются абсолютными. На самом деле это не так. Значения плотности пара часто неправильно интерпретируются в случаях, когда вещество выделяется при температуре окружающей среды, которая не позволяет им существовать в виде чистого газа или пара при нормальном атмосферном давлении.Неточный вывод может ввести респондентов в заблуждение относительно действий выпущенного газа или пара и может поставить под угрозу безопасность респондентов.

                                                                                                                                                                                                            Поскольку многие вещества (жидкости) имеют точки кипения значительно выше температуры окружающей среды, они не испаряются или не превращаются в чистые пары при выделении. (Чистые пары определяются как 100% концентрация над разливом). Этот момент важен, потому что плотности пара рассчитываются с использованием соотношений молекулярных масс, которые предполагают чистые пары. Более точным методом определения плотности пара вещества было бы сравнение смеси паров вещества в воздухе с паром чистого воздуха.Это соотношение позволит более точно определять плотность паров веществ. Чтобы это сделать, выполните следующие действия:

                                                                                                                                                                                                            Шаг 1. Рассчитайте приблизительную плотность чистого химического вещества при определенной температуре.
                                                                                                                                                                                                            pV = 1,3691 X молекулярная масса вещества, деленная на температуру (в Fo) + 460

                                                                                                                                                                                                            Шаг 2: Рассчитайте приблизительную плотность воздуха при температуре окружающей среды.
                                                                                                                                                                                                            pA = 39,566, деленное на температуру (в Fo) + 460

                                                                                                                                                                                                            Шаг 3: Вычислить относительную плотность паров воздушно-химической смеси.
                                                                                                                                                                                                            Относительная плотность пара = (C X pV) + [(100 — C) X pA], деленная на 100 X pA (C означает «насыщенную концентрацию» химического пара в воздухе в процентах по объему)

                                                                                                                                                                                                            * Если умножить 100 на давление пара вещества и разделить полученное произведение на 760, получится насыщенная концентрация.

                                                                                                                                                                                                            Хороший пример этой концепции — бензол. Бензол имеет молекулярную массу 78,1 а.е.м., исходя из формулы C6H6.Отношение плотности пара становится 2,69, если молекулярную массу бензола 78,1 разделить на молекулярную массу воздуха (29). Это соотношение указывает на то, что пары бензола будут накапливаться вблизи поверхности разлива и при высвобождении. В действительности пары были бы лишь немного тяжелее воздуха, если рассчитать по приведенным выше формулам.

                                                                                                                                                                                                            Давление паров бензола составляет 100 мм / рт. Ст. При 79 ° F, поэтому, умножив 100 мм / рт.Ответ 13,16%, что указывает на максимальную концентрацию паров бензола выше выброса при 79oF. Использование этих значений в приведенных выше уравнениях с допущением о температуре воздуха 79 ° F обеспечивает более точное значение плотности пара 1,22. Это значение означает, что смесь бензола и воздуха непосредственно над разливом бензола при температуре 79 ° F всего в 1,22 раза тяжелее воздуха, а не в соотношении 2,7, которое часто указывается как плотность пара для бензола.

                                                                                                                                                                                                            Эти уравнения и концепции используются в программных приложениях для диспергирования паров, чтобы лучше предсказать, где и как далеко паровые облака будут перемещаться при освобождении.Смесь с плотностью пара, близкой к плотности воздуха, будет быстро смешиваться с воздухом по мере удаления от разлива. Эта смесь не займет много времени, чтобы приблизиться к плотности воздуха и вести себя как нейтрально плавучая паровоздушная смесь. Смеси с отрицательной плавучестью будут вести себя как смеси тяжелее воздуха на большом расстоянии от разлива. Кроме того, смеси с положительной плавучестью будут вести себя как смеси легче воздуха.

                                                                                                                                                                                                            Точность такой информации поможет в оценке рисков и может изменить нашу тактику.В целях повышения личной безопасности рассмотрите возможность использования этих концепций и формул в своих будущих ответах. Как и во всех концепциях и теориях, есть исключения; обязательно сохраняйте бдительность и будьте осторожны! И теперь вы знаете остальную историю!

                                                                                                                                                                                                            Ресурс:

                                                                                                                                                                                                            • «Справочник по процедурам анализа химической опасности» Федерального агентства по чрезвычайным ситуациям (FEMA), Министерства транспорта США (DOT), Агентства по охране окружающей среды США (EPA)

                                                                                                                                                                                                            Плотность пара по Дюма: учебные заметки

                                                                                                                                                                                                            Плотность пара Дюма: учебные заметки

                                                                                                                                                                                                            Плотность пара Дюма

                                                                                                                                                                                                            Содержание : формулы, газовые законы, молярная масса

                                                                                                                                                                                                            Уровень : вводный

                                                                                                                                                                                                            Ссылка : J. Дюма, «Memoire sur quelques Points de la Théorie atomistique», Ann. Чим. Phys. 33 , 337-391 (1826) [упражнения 1-3]; J. Dumas, «Диссертация на тему Densité de la Vapeur de quelques corps simples», Ann. Чим. Phys. 50 , 170-8 (1832) [упражнение 4]

                                                                                                                                                                                                            Примечания : Жан-Батист Дюма (1800-1884) разработал метод определения молярной массы веществ, которые можно легко превратить в пары. Метод Дюма до сих пор является предметом лабораторных занятий на курсах химии.Он включает определение массы, объема, температуры и давления вещества в паровой фазе. При определении молярной массы в современном эксперименте по методу Дюма используется закон идеального газа; именно такой анализ используется в упражнениях. Во времена Дюма концепция родинки еще не была разработана. Он рассчитал молярные массы (или, скорее, относительные молекулярные массы) на основе относительных плотностей газа. Молярная масса пара относительно, скажем, водорода, была равна плотности пара, деленной на плотность водорода при сопоставимых условиях. Метод зависит от гипотезы Авогадро, а именно, что газы в сравнимых условиях содержат одинаковое количество молекул. [1] Дюма знал, как отрегулировать измеренную плотность, чтобы относиться к другому давлению (а именно, используя закон Бойля), и он знал, как приспособиться к другой температуре (а именно, используя закон Чарльза).

                                                                                                                                                                                                            В упражнении 4 рассматриваются данные Дюма о сере. Элементы, которые не образуют двухатомных молекул в газовой фазе (например, сера), и соединения с аномальным давлением пара (такие как NH 4 Cl и PCl 5 ) заставили Дюма усомниться в его методе и гипотезе, на которой он был основан.Дюма (и Авогадро) считали, что молекулы элементов в газовой фазе состоят из двух атомов. Многие другие химики считали, что газообразные элементы представляют собой отдельные атомы, которые, как и атомы, будут отталкивать друг друга, а не связываться друг с другом. Ожидали ли они одноатомных или двухатомных молекул, никто не ожидал и не принимал шестиатомные молекулы, такие как результаты Дюма для серы. Пары, связанные с хлоридом аммония и тому подобным, мы теперь распознаем как «давление паров диссоциации», которое изменяется с температурой и давлением не как идеальные газы, а как константы равновесия диссоциации.В любом случае работа Дюма была одной из немногих попыток интегрировать гипотезу Авогадро с химическими знаниями того времени. Полная интеграция подождет работы Канниццаро ​​еще через 30 лет. (Классические вычисления включают упражнения, основанные на работе Авогадро, Бойля и Канниццаро.)

                                                                                                                                                                                                            Теперь, конечно, вооружившись лучшим пониманием связывания и целым арсеналом инструментов для определения структуры, мы понимаем, что не существует единого агрегатного состояния для газофазных элементов.Большинство обычных элементов, которые при нормальных условиях являются газами, являются двухатомными (например, N 2 , O 2 , H 2 , Cl 2 ). Благородные газы (неизвестные во времена Дюма) одноатомны. Важной формой кислорода является трехатомный кислород, O 3 . Что касается серы, то наиболее стабильные формы кристаллической серы имеют молекулы S 8 . В газовой фазе S 8 и другие циклические структуры существуют в температурно-зависимом равновесии, которое включает S 2 и изолированные атомы серы (особенно при очень высоких температурах).Интересно, что все эксперименты Дюма с серой дали результаты, предполагающие S 6 в качестве среднего размера, но был некоторый разброс в зависимости от температуры.

                                                                                                                                                                                                            Педагогическое примечание : Обратите внимание на большое количество цифр, о которых сообщается, особенно в упражнении 2. Количественный анализ ошибок не был хорошо развит в начале 19 века, века, равно как и использование значащих цифр для выражения точности.

                                                                                                                                                                                                            Дополнительная информация : Подробный обзор основной первичной литературы по множественным пропорциям, атомной гипотезе и атомным весам, включая некоторую количественную обработку данных, можно найти у Леонарда Нэша, «Атомно-молекулярная теория», у Джеймса Брайанта Конанта, изд. , Гарвардские истории экспериментальной науки , т. 1 (Кембридж, Массачусетс: Гарвард, 1957), стр. 215-321.

                                                                                                                                                                                                            Решения : Чтобы загрузить решения, перейдите по адресу:
                                                                                                                                                                                                            http://web.lemoyne.edu/giunta/classicalcs/dumas.doc


                                                                                                                                                                                                            [1] Та же самая гипотеза была выдвинута Андре-Мари Ампером независимо от Авогадро. Дюма цитировал работы Ампера, а не Авогадро.
                                                                                                                                                                                                            Авторское право 2003 г., Кармен Джунта. Разрешается воспроизведение в некоммерческих образовательных целях.
                                                                                                                                                                                                            Вернуться к началу сайта Classic Chemistry

                                                                                                                                                                                                            Плотность пара — определение плотности пара в The Free Dictionary

                                                                                                                                                                                                            Например, если существует вероятность разлива или выброса химического вещества, плотность пара которого тяжелее воздуха, лучшее место для возвратной решетки будет на небольшой высоте (т. Е. При повышении температуры объемная доля жидкой воды уменьшается, а плотность водяного пара увеличивается. Увеличение емкости было нелинейной функцией плотности водяного пара с большим увеличением емкости выше 6 г / [м. sup.3] [H2] O (30-40% относительной влажности). Плотность пара кг / [м3] 115,6 41,9 82,4 Скрытое тепло кДж / кг 151,7 209,9 226,7 Насыщенное. На рисунке 1 (b) показано отслеживаемая температура воздуха и рассчитанная плотность водяного пара в соответствии с непосредственно измеренными температурой и относительной влажностью воздуха. Высокая плотность пара просто означает, что пары выхлопных газов тяжелее воздуха и имеют тенденцию опускаться или образовывать покров в емкостях, таких как газовые резервуары. может быть установлен в качестве входных данных при использовании алгоритма статистического поиска для профилирования плотности водяного пара.Установите двухчастотную модель поиска алгоритма линейной регрессии в качестве примера: предположение, вытекающее из уравнения 14, заключается в том, что средняя плотность водяного пара матрицы, [[rho]. Sub.w, m], и поверхностная плотность массы воды, [ [rho] ‘. sub.w, m], равны (т. е. различия в концентрации воды во внутренней матрице незначительны). Поскольку некоторые из расчетных областей на воздушной стороне могут быть двумерными, следует соблюдать осторожность при интерполяции значений параметры потока от мелкой сетки, на которой решались уравнения воздушного потока, до грубой сетки, на которой решались уравнения плотности пара и температуры.Если хлороформ, имеющий плотность пара 4,1, обнаружен возле поверхности воды в бассейне, если он не перемещается за счет турбулентности воздуха, то можно ожидать, что хлорорганические соединения с высокой плотностью пара ведут себя аналогичным образом. Вертикальная скорость ветра, температура, плотность водяного пара и т. Д. Концентрация [CO 2] и давление фильтровались через фильтр нижних частот с 8-полюсным активным фильтром Баттерворта 10 Гц для уменьшения наложения спектров и считывались регистратором данных (CR-10, Campbell Scientific, Логан, Юта) при 21,15 Гц . Некоторые из ключевых терминов, применимых к физическим данным, включают точки замерзания / плавления, точку кипения, температуру разложения, удельный вес, объемную плотность, pH, давление пара, растворимость в воде, летучие вещества, объемные проценты, скорость испарения, плотность пара. , молекулярная масса и коэффициент распределения масло / вода.

                                                                                                                                                                                                            Вопрос № c0a3d | Socratic

                                                                                                                                                                                                            Идея состоит в том, что вы можете использовать плотность пара газа, чтобы определить его молярную массу .

                                                                                                                                                                                                            Плотность пара рассчитывается путем рассмотрения того, сколько молекул газа могло бы занимать данный объем по сравнению с числом молекул газообразного водорода, # «H» _2 #, которые занимали бы тот же объем при тех же условиях для давление и температура.

                                                                                                                                                                                                            Проще говоря, плотность пара сообщает вам плотность газа относительно плотности газообразного водорода.

                                                                                                                                                                                                            Если вы воспользуетесь тем фактом, что масса газа, то есть количество молекул, которое он содержит, можно выразить через его молярную массу , вы можете сказать, что

                                                                                                                                                                                                            #color (blue) (| bar (ul (color (white) (a / a) «плотность пара» = «молярная масса газа» / «молярная масса H») _2color (white) (a / a) | ))) #

                                                                                                                                                                                                            В вашем случае считается, что неизвестный углеводород имеет плотность пара, равную # 35 #. (- 1) #

                                                                                                                                                                                                            Теперь вы знаете, что имеете дело с # «0.70-г «# образец этого газа #» X «#. Используйте его молярную массу, чтобы определить, сколько моль у вас есть в этом образце

                                                                                                                                                                                                            # 0,70 цвет (красный) (отмена (цвет (черный) («g»))) * «1 моль газа X» / (70 цвет (красный) (отмена (цвет (черный) («g»)))) = «0,010 моль X» #

                                                                                                                                                                                                            Утверждается, что этот образец содержит # «0,60 г» # углерода и

                                                                                                                                                                                                            #m_ «газ» = m_ (C) + m_ (H) #

                                                                                                                                                                                                            #m_ (H) = «0,70 г» — «0,60 г» = «0,10 г H» #

                                                                                                                                                                                                            Используйте молярные массы элементов, чтобы определить, сколько моль каждого у вас

                                                                                                                                                                                                            # «Для C:» 0.60 цвет (красный) (отмена (цвет (черный) («g»))) * «1 моль C» / (12color (красный) (отмена (цвет (черный) («g»)))) = «0,050 моль C «#

                                                                                                                                                                                                            # «Для H:» 0,10 цвет (красный) (отмена (цвет (черный) («g»))) * «1 моль H» / (1 цвет (красный) (отмена (цвет (черный) («g») ))) = «0,10 моль H» #

                                                                                                                                                                                                            Это количество молей каждого элемента в # 0,010 # моль газа # «X» #. Чтобы найти молекулярную формулу соединения , вам нужно выяснить, сколько молей каждого элемента содержится в # 1 # моль газа # «X» #.

                                                                                                                                                                                                            В вашем случае у вас будет

                                                                                                                                                                                                            # 1 цвет (красный) (отмена (цвет (черный) («родинка X»))) * «0,050 молей C» / (0,010 цвет (красный) (отмена (цвет (черный) («родинки X»))) ) = «5 моль C» #

                                                                                                                                                                                                            # 1 цвет (красный) (отмена (цвет (черный) («родинка X»))) * «0,10 молей H» / (0,010 цвет (красный) (отмена (цвет (черный) («родинки X»))) ) = «10 моль H» #

                                                                                                                                                                                                            Следовательно, молекулярная формула углеводорода

                                                                                                                                                                                                            # цвет (зеленый) (| bar (ul (цвет (белый) (a / a) «C» _5 «H» _10color (white) (a / a) |))) #

                                                                                                                                                                                                            CONCOA — Свойства хлористого водорода

                                                                                                                                                                                                            Хлористый водород (HCl)
                                                                                                                                                                                                            ОПИСАНИЕ (водород Хлорид):

                                                                                                                                                                                                            Безводный хлорид водорода представляет собой бесцветный, едкий едкий газ с удушающим запахом. Хлористый водород тяжелее воздуха и сильно дымится на влажном воздухе. Он хорошо растворяется в воде, спирте и эфире. Обычно Используется при производстве винила, хлопка, а также для травления полупроводниковых кристаллов.

                                                                                                                                                                                                            ФИЗИЧЕСКИЕ ПОСТОЯННЫЕ (водород Хлорид):

                                                                                                                                                                                                            Молекулярный вес: 36,461
                                                                                                                                                                                                            Давление пара при 70 ° F: 613 фунтов на кв. Дюйм изб.
                                                                                                                                                                                                            Удельный объем при 70 ° F., 1 атм .: 10,6 куб. Футов / фунт
                                                                                                                                                                                                            Точка кипения при 1,0 атм. : -121,05 ° F
                                                                                                                                                                                                            Температура замерзания при 1 атм. : -173,54 ° F
                                                                                                                                                                                                            Удельный вес, газ при 32 ° Ф., 1 атм. (Воздух = 1): 1.268
                                                                                                                                                                                                            Плотность, газ при 0 ° C, 1 атм .: 1,639 г / л
                                                                                                                                                                                                            Плотность, газ при -36 ° C 1,194 г / мл
                                                                                                                                                                                                            Критическая температура: 124,5 ° F
                                                                                                                                                                                                            Критическое давление: 1198 фунтов на квадратный дюйм
                                                                                                                                                                                                            Критическая плотность: 0. 42 г / мл
                                                                                                                                                                                                            Скрытая теплота испарения @ б.п.: 103,12 кал / г
                                                                                                                                                                                                            Скрытая теплота плавления @ т.пл .: 13,05 кал / г
                                                                                                                                                                                                            Удельная теплоемкость, газ при 15 ° C, 1 атм. C p : 0,1939 кал / (г) (° C)
                                                                                                                                                                                                            Удельная теплоемкость, газ при 15 ° C., 1 атм. C v : 0,1375 кал / (г) (° C)
                                                                                                                                                                                                            Удельная теплоемкость, газ @ 15 ° C, 1 атм., C p / C v : 1,41
                                                                                                                                                                                                            Растворимость в воде при 0 ° C, 1 атм. : 82,31 г / 100% (г) H 2 O
                                                                                                                                                                                                            Диэлектрическая проницаемость, жидкость @ -85.0 ° С .: 6,35
                                                                                                                                                                                                            Поверхностное натяжение при -92,9 ° C: 24,72 дин / см
                                                                                                                                                                                                            Вязкость, газ при 20 ° C, 1 атм. : 0,0156 сантипуаз
                                                                                                                                                                                                            Теплота образования, газ @ 25 ° С .: -22,14 ккал / моль

                                                                                                                                                                                                            СОЕДИНЕНИЯ ЦИЛИНДРОВ (водород Хлорид):

                                                                                                                                                                                                            Стандартное соединение цилиндра CGA V-1: CGA 330
                                                                                                                                                                                                            Стандартное соединение цилиндра CGA V-1 Сверхвысокая надежность: CGA 634

                                                                                                                                                                                                            CONCOA — Свойства сероводорода

                                                                                                                                                                                                            Сероводород (H 2 S)
                                                                                                                                                                                                            ОПИСАНИЕ (водород Sulfide):

                                                                                                                                                                                                            Сероводород — бесцветный горючий газ с неприятный запах. Он считается токсичным газом, перевозимым в стальных баллонах в качестве сжиженный газ при собственном давлении пара примерно 250 фунтов на кв. дюйм при температуре 70 ° F. используется как реагент для химического анализа, добавка к маслам и при приготовлении сульфиды металлов.

                                                                                                                                                                                                            ФИЗИЧЕСКИЕ ПОСТОЯННЫЕ (водород Сульфид):

                                                                                                                                                                                                            Молекулярный вес: 34,08
                                                                                                                                                                                                            Давление пара при 70 ° F.: 252 фунтов / кв. Дюйм изб.
                                                                                                                                                                                                            Удельный объем при 70 ° F, 1 атм .: 11,23 куб.фут / фунт
                                                                                                                                                                                                            Точка кипения при 1,0 атм. : -76,6 ° F
                                                                                                                                                                                                            Температура замерзания при 1 атм. : -121,9 ° F
                                                                                                                                                                                                            Тройная точка при насыщении давление: -113.4 ° F
                                                                                                                                                                                                            Удельный вес, газ при 15 ° С, 1 атм. (Воздух = 1): 1,1895
                                                                                                                                                                                                            Плотность, газ при 0 ° C, 1 атм .: 1,5392 г / л
                                                                                                                                                                                                            Плотность, жидкость @ b.p ..: 0,993 г ​​/ мл
                                                                                                                                                                                                            Критическая температура: 212. 7 ° F
                                                                                                                                                                                                            Критическое давление: 1309 фунтов на квадратный дюйм
                                                                                                                                                                                                            Критическая плотность: 0,349 г / мл
                                                                                                                                                                                                            Скрытая теплота испарения @ б.п.: 4,463 ккал / моль
                                                                                                                                                                                                            Скрытая теплота плавления @ т.пл .: 568 кал / моль
                                                                                                                                                                                                            Пределы воспламеняемости в воздухе: 4.3-45% к объему
                                                                                                                                                                                                            Температура самовоспламенения: 500 ° F
                                                                                                                                                                                                            Удельная теплоемкость, газ при 25 ° C, 1 атм. C p : 0,240 кал / (г) (° C)
                                                                                                                                                                                                            Удельная теплоемкость, газ при 25 ° C, 1 атм. C v : 0,181 кал / (г) (° C)
                                                                                                                                                                                                            Удельная теплоемкость, газ @ 25 ° С., 1 атм., C p / C v : 1,32
                                                                                                                                                                                                            Теплопроводность, газ @ 0 ° С: 3,04 x 10 -4 кал / (сек) (см 2 ) (° C / см)
                                                                                                                                                                                                            Вязкость, газ при 32 ° F, 1 атм. : 0,01166 сантипуаз
                                                                                                                                                                                                            Растворимость в воде при 0 ° C. , 1 атм. : 0,672 г / 100 мл H 2 O

                                                                                                                                                                                                            СОЕДИНЕНИЯ ЦИЛИНДРОВ (водород Сульфид):

                                                                                                                                                                                                            Стандартное соединение цилиндра CGA V-1: CGA 330
                                                                                                                                                                                                            Стандартное соединение цилиндра CGA V-1 Сверхвысокая надежность: CGA 722

                                                                                                                                                                                                            Связь между плотностью пара и относительной молекулярной массой

                                                                                                                                                                                                            Относительная молекулярная масса газа или пара — это отношение массы одной молекулы газа или пара к массе одного атома водорода.

                                                                                                                                                                                                            ОТНОШЕНИЯ МЕЖДУ ПЛОТНОСТЬ ПАРА И ОТНОСИТЕЛЬНАЯ МОЛЕКУЛЯРНАЯ МАССА

                                                                                                                                                                                                            1. Относительная молекулярная масса: (водородная шкала)

                                                                                                                                                                                                            Относительная молекула Масса газа или пара — это соотношение между массой одной молекулы газ или пар массой одного атома водорода.

                                                                                                                                                                                                            2.Плотность пара:

                                                                                                                                                                                                            Плотность пара — это отношение массы определенного объема газа или пара к массе равный объем водорода, измеренный при одинаковых условиях температуры и давление.

                                                                                                                                                                                                            Плотность пара (V.D.) =


                                                                                                                                                                                                            По закону Авогадро равные объемы всех газов содержат одинаковое количество молекул.

                                                                                                                                                                                                            Итак, пусть количество молекул в одном объеме = n, тогда

                                                                                                                                                                                                            В.D. при S.T.P =


                                                                                                                                                                                                            Отмена «n», обыкновенно, вы получите В.Д.


                                                                                                                                                                                                            При сравнении формула плотности пара с относительной молекулярной массой, их можно представить как В.Д.


                                                                                                                                                                                                            Поэтому вы можете подставьте приведенное выше уравнение в уравнение 7.1 и получите следующую формулу


                                                                                                                                                                                                            Сейчас на кресте умножение, у вас

                                                                                                                                                                                                            2 × плотность пара = Относительная молекулярная масса газа

                                                                                                                                                                                                            (или)

                                                                                                                                                                                                            Относительная молекулярная масса = 2 × плотность пара

                                                                                                                                                                                                            Учебные материалы, примечания к лекциям, задания, ссылки, объяснение описания Wiki, краткие сведения

                                                                                                                                                                                                            .